Comments

  • What can I know with 100% certainty?
    ↪Gnomon
    Can you show me the calculation, please?
    Truth Seeker

    The linked article in my post describes the procedure for calculating the statistical probability of a personal belief. Other than objective evidence or blind faith, that may be as close to "certainty" as you can get to logical truth. Because it begins with a subjective guess, the calculation will never produce 100% certainty. Here's Bayes' formula in the form of an equation. :smile:


    bayes__theorem.svg
  • What can I know with 100% certainty?
    Thank you for your reply "flannel jesus." How would I calculate what percentage of certainty I assign to things such as the objective existence of my body, other humans, non-human organisms, the Earth and the rest of the universe?Truth Seeker
    In the 18th century, Thomas Bayes developed a method for quantifying Certainty : it's called "Statistics". :smile:

    Bayesian probability :
    Broadly speaking, there are two interpretations of Bayesian probability. For objectivists, who interpret probability as an extension of logic, probability quantifies the reasonable expectation that everyone (even a "robot") who shares the same knowledge should share in accordance with the rules of Bayesian statistics, which can be justified by Cox's theorem.[3][10] For subjectivists, probability corresponds to a personal belief.
    https://en.wikipedia.org/wiki/Bayesian_probability
  • A first cause is logically necessary
    ...as I've learned from Gnomon, causation is believed but not yet proven. — ucarr

    I would questionwhat you mean by 'not proven'. Without causation all of science and reason goes out the window. If causation is gone, then I can't say you typed your reply to me. "You" didn't cause it. And that's absurd. — Philosophim

    Perhaps Gnomon can elaborate so rules of inference governing formal proofs not yet satisfied by reasoning about causation.
    ucarr
    Hey, I'm just accepting David Hume's reasoning, about the universality of cause & effect. I'm not an expert in these matters, so you can argue with him.

    Today, we associate the word "Energy" with physical changes in the world. Yet it is defined, not as a physical thing, but as an "ability" or "capacity" or "efficacy" or "potential" which are no more empirically provable than "causation". Personally, I take predictable physical determinism for granted, for pragmatic reasons, and make no attempt to prove it, logically or empirically. I assume that's what Hume meant by "certitude". :smile:

    David Hume & the Theory of Causation :
    Causation describes a cause-and-effect relationship, where one thing causes another to occur. However, Hume argued that causation is not always empirically sound because it cannot be proven because experiences are subjective and flawed.
    https://study.com/academy/lesson/the-metaphysics-of-causation-humes-theory.html

    David Hume: Causation
    Once more, all we can come up with is an experienced constant conjunction. Of the common understanding of causality, Hume points out that we never have an impression of efficacy. Because of this, our notion of causal law seems to be a mere presentiment that the constant conjunction will continue to be constant, some certainty that this mysterious union will persist. Hume argues that we cannot conceive of any other connection between cause and effect, because there simply is no other impression to which our idea may be traced. This certitude is all that remains.
    https://iep.utm.edu/hume-causation/

    Is it ever possible to actually 'prove' causation? :
    You can prove 'causation' with respect to one context and one event but never on a universal canvas of time where you can explain all things at the same time. You will necessarily make some assumptions. So proof of 'causation' will come with that baggage of unexplained assumptions.
    https://www.quora.com/Is-it-ever-possible-to-actually-prove-causation
  • A first cause is logically necessary
    What I presented comes from Deacon.ucarr
    The phrase --- "Do you reject the belief causation resides within dynamical systems of self-organization phase-shifted across ascending levels of organization towards effects?" --- is over my head. So I can't agree or deny. If you say so, I'll assume it makes sense to Deacon. :smile:

    Is it correct to say you see causation -- structurally speaking -- as a generalization in parallel with the specific energy-and-change relationship with respect to an invisible agent that causes transformation from one state-of-being to another state-of-being?ucarr
    Again, above my pay grade. But yes, as I understand Causation, the agent of "transformation" is invisible, and is knowable only by inference from observations of state A (before) and state B (after) the physical changes noted. The "invisible agent" is called by various names by scientists : energy, inertia, mass, photon, potential, etc. When a cue ball hits a stack of billiard balls, some unseen something seems to have been transferred from the moving cue ball to the stationary eight ball. We still don't know what-it-is, in material terms, but we do know a lot about what it does, its physical actions & reactions.

    In my personal thesis, I follow the implications of cutting-edge physics --- which is also over my head --- to draw philosophical inferences about Causation, Transformation, and Information. The term "information" originally applied only to ideas in a human mind. But now it is being used to describe all kinds of Transformations*1. Physicists tend to think of physical Energy as fundamental, but some philosophers view Information as the primitive of Causation*2. Studies of complex systems, such as biological entities, have been enhanced by treating Information as an analog of Energy*3. So you can call that "invisible agent of transformation" Energy or Causation or Information or Spirit, depending on the context, and your own proclivities. :nerd:


    *1. Does Energy = Information? :
    Energy is a mysterious force that causes things to move. Energy is not information but it is required to transfer information
    https://www.physicsforums.com/threads/the-nature-of-energy-a-philosophical-perspective.122587/

    *2. What is information? :
    Information philosophy extends that study to the communication of information content between material objects, including how it is changed by energetic interactions with the rest of the universe.
    https://www.informationphilosopher.com/

    *3. Complexity, Entropy & the Physics of Information :
    They connect the natural sciences to the science of computation, and they characterize the emergence of classical physics from the quantum realm in the early universe.
    https://www.sfipress.org/news/complexity-entropy-and-the-physics-of-information
  • A first cause is logically necessary
    I'm recalling from memory you citing Hume re: causation. The gist of your point is that causation, in his view, is an inference from observed patterns of apparently connected changes in states of being within the world. His conclusion, as reported by you, states that the concept of causation stands upon empirically-derived impressions of the world. In closing, you said these impressions are generally understood to fall short of a proof of the concept of causation.ucarr
    I would prefer that you quote the assertions you are responding to. I don't remember exactly how I worded the comments on Hume's causation. But I wouldn't say that "the concept of causation stands on empirically-derived impressions". Empirical typically implies recorded & confirmed scientific evidence. But up until Hume's day, the notion of Causation (by some invisible entity) was taken-for-granted by most people, as a reasonable-but-untested inference from sensory observations. Therefore, Hume was philosophically & scientifically critical of that presumption.

    The connection between sequential causal events (what we now call Energy) was invisible & intangible. There was no discernible difference between the putative "cause" and the presumed "effect". And 17th century Natural Philosophy had no formal concept of Energy, but the ancient notion of Spirit persisted. So he, not I, said the commonsense belief in Causation --- perhaps as a manifestation of heavenly Spirit acting in the world --- "falls short of" empirical proof. Where Kant spoke of "consult not experience", I'm guessing he was referring to what we now call Empirical Science. :smile:


    David Hume & the Theory of Causation :
    Hume's theory of causation states that causality is formed from the relationship between two impressions or ideas in the mind. However, because knowledge comes from experiences, assumptions of causality are intrinsically flawed and cannot be proven.
    https://study.com/academy/lesson/the-metaphysics-of-causation-humes-theory.html

    Kant and Hume on Causality :
    And as the first imagination or invention of a particular effect, in all natural operations, is arbitrary, where we consult not experience; so must we also esteem the supposed tye or connexion between the cause and effect, which binds them together, and renders it impossible that any other effect could result from the operation of that cause. . . .
    Thus, although Kant does not explicitly mention Hume in Dreams of a Spirit-Seer, the parallels with Hume’s Enquiry are striking indeed

    https://plato.stanford.edu/entries/kant-hume-causality/


    Do you reject the belief causation resides within dynamical systems of self-organization phase-shifted across ascending levels of organization towards effects?ucarr
    Could you rephrase that question in more conventional terms? Or explain your terms in more detail. For the record, I don't deny Causation; but I do think it's a mental inference, not a spiritual force, in the world. Instead, the term Energy now covers physical actions that used to be attributed to Spirits.
  • Is philosophy just idle talk?
    If we apply Hegel's idea to philosophy at large, it is not idle talk at all but the necessary ingredient for a dynamic development of ideas.Pez
    Yes, but not just ideas. The point of Hegel's (Plato's) dialectic is that logical & physical contradictions (competing ideas & forces) --- in isolation --- are not just false, but stagnant*1. Yet contradictions, when synthesized by physical stresses or social debate or philosophical dialogue, can be progressive. In fact, some thinkers have concluded that all emergent evolutionary processes are dialectic in method*2.

    Physical evolution opposes Positive & Negative forces that push & pull on matter, resulting in adaptive changes in constitution. Likewise, Metaphysical evolution opposes contrary ideas & beliefs, in order to adapt them to new situations*3. Hence, in effect, synthetic Philosophy is metaphysical evolution, winnowing & harmonizing obsolete beliefs, in order to get us Closer to Truth*4. :smile:

    *1. Dialectical Evolution of Truth :
    Dialectic in Classical philosophy is denoted as a form of discussion that takes place between two entities; it is the logical reasoning and a form of a method through which the introduction of proposition and counter proposition is practised while the main aim of the debate remains the same, that is reaching an objective truth through this course. Much of the prestige and role is deserved by the classical philosophers Socrates and Plato in the evolution of the Dialectic method.
    https://unacademy.com/content/upsc/study-material/philosophy/dialectical-method/

    *2. Physical & Historical Evolution are Dialectical Processes :
    Darwinism is Hegelian dialectics applied to biology — or you might say that Hegelian dialectics is Darwinism applied to history.
    https://evolutionnews.org/2020/09/darwinism-as-hegelian-dialectics-applied-to-biology/

    *3. Evolution of Truth :
    Our understanding of the truth is constantly evolving.
    https://evolutionoftruth.com/

    *4. Both/And Principle :
    My coinage for the holistic principle of Complementarity, as illustrated in the Yin/Yang symbol. Opposing or contrasting concepts are always part of a greater whole. Conflicts between parts can be reconciled or harmonized by putting them into the context of a more complete whole system.
    https://blog-glossary.enformationism.info/page10.html
  • A first cause is logically necessary
    A first cause is self-existent though. I think that's the problem he has. He doesn't like the idea that there was nothing, then something. What I'm trying to show him is that an eternally self-existent thing is no different. There is nothing which explains its being. No limitations on what could have been besides the fact of its existence.Philosophim
    According to the worldview of Materialism, "nothing" is non-sense. And, since the physical world does exist, it must have always existed in some form or other. Also, how or why it came to be is not an empirical question, hence more non-sense. If there is nothing to explain its existence, then it's provenance is a matter of Faith, or Reason.

    Ancient Materialism (e.g. Atomism) was a hypothetical solution to a philosophical question. But sensible modern Materialism seems to be primarily an alternative to religious answers to "Why?" questions*1. Apparently, it assumes that philosophy is impotent (decorative) to answer any questions about Reality. Hence Ideal notions, such as "something from nothing", are literally nonsensical, since we cannot sense nothingness. And from the perspective of modern Materialism, non-sensible is non-sensical.

    Although you seem to be trying to evade the implication of "spiritual beings", by limiting the discussion to logical reasoning, not religious doctrine, even your First Cause is --- by definition of Materialism --- un-real, and non-sensible, therefore implausible. In Materialism, what is Real, is what is sensible*2.

    Ironically, modern science postulates several causal features of reality that are logical inferences instead of sensory observations. For example Energy is the universal cause of all changes in the world, but we never detect the Energy per se, we only infer its logically-necessary existence from after-effects in material objects. Likewise, the notion of electric or quantum Fields is a logical inference from observation of changes in the material world*3. How that universal or local field came to be --- "popped into existence" --- is irrelevant for pragmatic Science : it just is, and it works.

    Those invisible and intangible features of Reality, are accepted because they allow us to predict physical behavior. But most of those predictions are logical extrapolations from known rules of Nature. And how do we know those regulations of physical behavior? By rational inference, as expressed in terms of a> philosophical Epistemology, or b> scientific Natural Laws, or c> religious Supernatural beliefs. None of which are empirical observations, but unlike First Cause, some do have practical applications in the Real world.

    The First Cause is simply another inference from logical necessity. But is it Real? Of course not. It's Ideal. A belief, not a fact; just like an unexplainable quantum Field --- Scientists like to think it's a fact, "because it works". The Prime Mover only works in the beginning of world-making, not during its mundane operations. Like Plato's world-creating Craftsman, the First Cause does its work, then disappears into the work itself as ongoing Causation : e.g. Energy. :smile:


    *1. What is the metaphysical theory of materialism?
    In general, the metaphysical theory of materialism entails the denial of the reality of spiritual beings, consciousness and mental or psychic states or processes, as ontologically distinct from, or independent of, material changes or processes.
    https://www.rep.routledge.com/articles/thematic/materialism/v-1

    *2. Sensible :
    a> based on or acting on good judgment and practical ideas or understanding
    b> practical and functional rather than decorative.


    *3. Is the Quantum Field real? :
    For generations, physicists argued whether those quantum fields were actually real, or whether they were simply calculational tools. Nearly a full century later, we're certain that they're real for one unambiguous reason: they carry energy.
    Note --- Both QF and Energy are logical inferences, not observations

    Provenance :
    a> the place of origin or earliest known history of something.
    b> the beginning of something's existence; something's origin.


  • A first cause is logically necessary
    A lot here but very interesting. It does remind of Hume's argument that we do not observe causation, just a sequence of events, but we impose it on to the experience, more or less conceptually.Gary Venter
    Gravity (pulling action without material connection) was a mystery to Newton, and a mathematical/logical concept to Einstein. His immaterial notion of causation may be related to his incredulous "spooky action at a distance" characterization of quantum entanglement, which involves sharing Information. In the 21st century, scientists have correlated Causal Energy with Knowable Information*1. :smile:

    *1. A proposed experimental test for the mass-energy-information equivalence principle :
    A recent conjecture, called the mass-energy-information equivalence principle, proposed that information is equivalent to mass and energy and exists as a separate state of matter.
    https://pubs.aip.org/aip/sci/article/2022/9/091111/2849001/A-proposed-experimental-test-for-the-mass-energy


    Physics has no theory of how matter can warp space-time. . . . the quantum field is what is fundamental and that in itself actually creates space through informational effects.Gary Venter
    What is warped by Gravity is not just space, but Space-Time, which seems to include everything in the universe ( Space : matter ; Time : energy ). The quantum fields (17 types???) are not material objects, but Mathematical/Logical relationships between infinite "quantum oscillators" --- whatever that is. For my own philosophical purposes, I imagine the Quantum Field as Aristotle's eternal Potential, which when Actualized into space & time (matter & energy) becomes everything in the world that we can sense (i.e. Reality). :nerd:


    a lump of matter is built up from something pretty non-material. . . . they are waves of informationGary Venter
    Einstein's E=MC^2 equation related invisible Energy and mathematical Mass to a dimensionless logical constant, which together we humans experience as Matter. Plato also related "rational principles" (Forms) with the creation of physical matter. Was he on to something, that took centuries to be expressed in a simple T-shirt equation? :joke:


    conscious potential . . . the essential nature of matter and consciousness are the sameGary Venter
    Your comments remind me of my own Information-based philosophical worldview. We seem to be thinking along the same lines.

    My personal thesis, Enformationism, postulates that the essence of physical Matter & Energy, and of metaphysical Consciousness, is the Principle of Potential that I call EnFormAction (power to give form to the formless). That's similar to Plato's First Cause (Form), and Aristotle's Prime Mover (Energy).

    I'd like to compare notes, to see where you got your ideas about Quantum Physics & Information. Mine probably originated in John A. Wheeler's 1989 "it from bit" conjecture. :grin:
  • A first cause is logically necessary
    Let me give you an example. Lets say that space has always existed. What caused space to always exist? Nothing. Therefore space is a first cause. It is something within causality that itself was not caused. So no, an eternal universe does not preclude a first cause. Why is the universe eternal opposed to not? What caused it to be that way? Nothing.Philosophim
    seems to be reasoning from the assumption that the physical universe --- space-time, matter, energy --- could possibly be self-existent, hence no need for a First Cause or Creator. If so, the postulated Multiverse would be either infinite in parallel, in the sense of Many Worlds, or a serial re-incarnation of a single self-existent 'Verse.

    On the other hand, ancient thinkers, such as Plato, Aristotle, Taoists, and Torah writers, seemed to assume that a perishable world (Death follows Life) of serial causation could not be self-existent*1. Even Plato's less-than-perfect workman --- demiurge --- follows a pre-existing plan. So, they reasoned that some non-perishable outside force or principle was necessary to jump-start the world of cycling causes & effects*2. Some postulations for The Eternal Creative Principle were : Forms, Logos, Tao, and the nameless eternal creator of Genesis*3. :smile:


    *1. Craftsman vs Creator :
    ‘The nature of things’ examines Plato's understanding of the natural world. In the Timaeus, Plato describes the creation of the world as work done by a divine Craftsman, who does the job by reference to a model — a system of rational principles. The real world is not, as we uncritically take it to be, the world around us that our senses report to us; it is rather what we grasp in thought when exercising our minds in abstract philosophical argument, in particular arguments which lead to what Plato calls Forms — the Forms which function as patterns for the Craftsman as he makes our world. . . .
    Plato's God is a workman who does the best he can with the materials he has to work with; he creates order from chaos, but he does not create the original materials from nothing.

    https://academic.oup.com/book/391/chapter-abstract/135200895?redirectedFrom=fulltext
    Note --- In this scenario the demiurge obtained his materials from the principle of Chaos, which was merely Generalized Potential (Causality), but not organized into Matter or Energy. So the Prime Cause must include both Potential (power, energy) and Plan (laws, designs). Neither of which correspond to the mundane energy & matter of the physical world.

    *2. On the First Cause :
    "Our Stoic philosophers, as you know, declare that there are two things in the universe which are the source of everything — namely, cause and matter. Matter lies sluggish, a substance ready for any use, but sure to remain unemployed if no one sets it in motion. Cause, however, by which we mean reason, moulds matter and turns it in whatever direction it will, producing thereby various concrete results. Accordingly, there must be, in the case of each thing, that from which it is made, and, next, an agent by which it is made. The former is its material, the latter its cause." ___Seneca, Stoic philosopher
    https://monadnock.net/seneca/65.html

    *3. Nameless First Cause :
    The Tao [Way] that can be told of is not the eternal Tao; The name that can be named is not the eternal name. The Nameless is the origin of Heaven and Earth; The Named is the mother of all things.
    https://www.oxfordreference.com/display/10.1093/acref/9780191826719.001.0001/q-oro-ed4-00006490
  • A first cause is logically necessary
    I'm not sure about the universe as a whole, but physics is giving up on causation in the physical sense - a lot now is more like logical causation. In general relativity, a big mass, like the Earth, curves space, and that gives the illusion of a force of gravity, but there is no such force in physics anymore. And there is no mechanism for how the mass manages to curve space. Similarly in quantum mechanics, the Schrodinger wave equation does not let the electrons in an atom collapse into the protons in the nucleus, even though no force is created to oppose the electrical attraction pulling them in. How does an equation accomplish that? Sounds more like logical causation to me.Gary Venter
    Exactly! What we call "Causation" is not a physical or mechanical Force, but a logical inference from observation of sequential physical changes.

    Your post metaphysically caused me to question how Matter can produce the sensed Effect we call "Gravity". Einstein's E=MC^2 formulation implies that the Energy constituting a physical object can be transformed (somehow) into intangible mathematical Mass, which we sensibly experience as Gravity. Similarly, the Brain (somehow) processes neural Energy into the metaphysical Experience we call "Mind". Chalmers was posing an equivalent question of Causation in asking how a lump of matter could produce the intangible-but-sensible*1 effect (feeling) we call "Mind". Is it magic, or physics, or metaphysics?

    For the same reason, I conclude that the First Cause of the on-going changes in our world is not a physical force, but a metaphysical Potential with Actual physical effects. Aristotle provided a philosophical definition of the "Potential" principle*2. :smile:


    *1. Sensible :
    Sensible heat is literally the heat that can be felt.
    https://energyeducation.ca/encyclopedia/Sensible_heat

    *2. Aristotle's Potential Principle :
    Matter is the potential factor, form the actualizing factor. (Aristotle further posited the existence of a prime mover, or unmoved mover, i.e., pure form separate from matter, eternal and immutable.)
    https://www.britannica.com/topic/potentiality
    Note --- Today we might call that "actualizing factor" of Causation the Power to Enform, i.e. Information. Matter is the substance acted upon, but EnFormAction is the cause of all changes.
  • A first cause is logically necessary
    I'm asking if work towards finding a proof is more appropriate for the philosopher than for the scientist.ucarr
    What kind of "proof" --- for a "metaphysical issue" --- would you expect to find, as a philosopher? Can we send a philosophical space-probe back in time to find the empirical First Cause of the Cosmos? Can a valid logical argument prove the truth (existence/reality) of a metaphysical belief?

    Mathematical "proof" is simply logical consistency (derived from true premises), and Empirical "proof" is based on abundance of non-contradictory physical evidence. Philosophical "proof", though, is a logical argument that is accepted as true in a convinced mind. But when two philosophers argue, the discussion can go-on indefinitely, without reaching a mutually satisfactory conclusion. So, I would say that finding slam-dunk proof --- for a Prime Mover --- is in-appropriate for mathematicians & scientists & philosophers.

    For example, in Science Ideated, Bernardo Kastrup argues with two philosophers on the metaphysical existence of Consciousness : a process or qualia, not a physical thing. One argues that "consciousness doesn't happen, it's a mistaken construct", And the other says, "our introspective systems monitor these neural processes but misrepresent them as a simple quality". Have any of these thinkers proven anything? Is your belief or disbelief in your own personal Awareness based on objective evidence, or on personal experience --- the very non-thing in question? :smile:

    PS___ First Cause arguments are mathematical/logical in that they are true only if the premises are true. Unfortunately, in this special case of reasoning, the First Cause is also the Premise in question.


    Can you prove anything in philosophy?
    It's a common misconception that you can prove whatever you like in philosophy. In reality, it's hard to find completely convincing arguments which conclusively establish some conclusion.
    https://www.quora.com/Can-you-prove-anything-in-philosophy

    Is it ever possible to really prove anything in philosophy? :
    Richard Rorty saw philosophy as a tool that you used to solve problems. He agreed that you couldn't definitively prove things in philosophy
    https://www.reddit.com/r/askphilosophy/comments/12rjpo9/is_it_ever_possible_to_really_prove_anything_in/

    What is the best evidence or proof for metaphysics? :
    Metaphysics means what is “after” or “beyond” physics. So almost by definition, is outside the domain of physical evidence.
    https://www.quora.com/What-is-the-best-evidence-or-proof-for-the-existence-of-metaphysics
  • A first cause is logically necessary
    Do you assess this lack of proof as a metaphysical issue?ucarr

    Only if it leads to false beliefs. Most of the time, correlation is "close enough for government work". But technically, in logic and statistics, conflation of correlation with causation is a fallacy. In complex situations, other factors may disguise the "true" cause {see stork/baby image}. :smile:

    Gnomon Reply to LFranc in this thread :
    "Therefore, in this thread, we are ultimately arguing about a metaphysical principle*2 to explain all changes in the world. So, if we track all physical changes back to the Big Bang, and stop, we have merely defined the First Effect, not the First Cause."

    Correlation often, but not always, implies Causation :
    It is certain, that not only in philosophy, but even in common life,we may attain the knowledge of a particular cause merely by one experiment, provided it be made with judgment, and after a careful removal of all foreign and superfluous circumstances. Now as after one experiment of this kind, the mind, upon the appearance either of the cause or the effect, can draw an inference concerning the existence of its correlative; and as a habit can never be acquired merely by one instance; it may be thought, that belief cannot in this case be esteemed the effect of custom. ___David Hume
    https://stats.stackexchange.com/questions/551312/who-first-coined-the-phrase-correlation-does-not-imply-causation

    7714231-Randall-Munroe-Quote-Correlation-doesn-t-imply-causation-but-it.jpg

    f1f6cafb-6444-453f-a5c5-8f603ab41a23_SP+562+-+Correlation+is+not+causation.png?auto=compress,format
  • A first cause is logically necessary
    I think you missed the point of my philosophical distinction between inferred Belief (certitude) in a Cause, and a scientifically-proven Fact of the Agent (bacteria) of an Effect (meningitis). The footnote gave the context*1. David Hume defined the concept of Causation as an inferred mental relationship, not a physical thing*2. — Gnomon


    You and Hume characterize causation as deduction?
    ucarr
    Well, technically, Deduction from data*1 is just one way to understand Change in the world. It begins with observation of a general principle ((transformation ; metamorphosis) and subtracts (abstracts) everything that is not consistently associated with observed Effects. When Induction and Abduction also agree on the Deduction, we can be pretty sure that the Cause and Effect are correlated by some transformational Principle, that we call "Causation" --- or in some cases "Agency". And yet, due to the limitations of data and reasoning, mere Correlation of variables does not prove Causation. We could be missing something.

    Aristotle called that agent of change "Morphe" (form). Other ancient thinkers used the term "Spirit" to label that invisible causal Agency, but moderns tend to use the term "Energy" or "Power". Now, in the 21st century, another term has been applied to describe physical Transformation : Information*2 --- or as I call it in my thesis : EnFormAction*3 (the power to transform). The bottom line here, is that Causation is not something we can see or touch or dissect physically. But we can analyze it by means of Reason. Hence, the agency of transformation must be rationally inferred or deduced. :smile:

    *1. Deduction of Causation :
    There are three types of causal reasoning: deduction, induction, and abduction. Deduction is the use of data and arguments to come to a guaranteed conclusion. Induction is when conclusions are drawn based on a limited set of data.
    https://study.com/academy/lesson/causal-reasoning-definition-examples.html

    *2. Energy :
    Scientists define “energy” as the ability to do work, but don't know what energy is. They assume it's an eternal causative force that existed prior to the Big Bang, along with mathematical laws. Energy is a positive or negative relationship between things, and physical Laws are limitations on the push & pull of those forces. So, all they know is what Energy does, which is to transform material objects in various ways. Energy itself is amorphous & immaterial. So if you reduce energy to its essence of information, it seems more akin to mind than matter.
    https://blog-glossary.enformationism.info/page8.html

    *3. EnFormAction :
    Ententional Causation. A proposed metaphysical law of the universe that causes random interactions between forces and particles to produce novel & stable arrangements of matter & energy.
    https://blog-glossary.enformationism.info/page8.html
  • A first cause is logically necessary
    I prefer the Hartle-Hawking No Boundary Conjecture² instead.180 Proof
    I can see why Hawking's spherical universe "conjecture" fits your Immanent belief system better than the Big Bang theory's exploding pin-point (Singularity) imagery. To each his own.

    I suspect Hawking was merely echoing Einstein's god-like description of the universe --- as seen from outside --- imagined as a four-dimensional sphere : "finite" in volume, but "unbounded" in topology. That imagery suited his mathematical purposes, just as the counter-intuitive Block Time (eternalism) model illustrated his concept that our perception of dualistic Space (matter) & Time (energy) is merely a common-sense interpretation of monistic Space-Time. But, my layman's philosophical purpose & preference is different from Einstein's genius mathematical intention. :smile:

    GOD'S {and Einstein's} VIEW OF HIS FINITE BUT UNBOUNDED CREATION
    sphere.jpg

    COSMOLOGIST'S MODEL OF BIG BANG
    4.jpg?ip=x480
  • A first cause is logically necessary
    After going to the doctor with mild symptoms, you're told your spinal column is infected with pneumococci bacteria. Since it's believed this infection causes spinal meningitis, you're advised to immediately undergo an aggressive program of antibiotics within the intensive care unit. Explain why you wouldn't dismiss this diagnosis as uncertain causal-belief-not-fact and go home untreated, or would you go home? Would you go home untreated, betting on fact-based-mind-over-uncertain causal-belief?ucarr
    I think you missed the point of my philosophical distinction between inferred Belief (certitude) in a Cause, and a scientifically-proven Fact of the Agent (bacteria) of an Effect (meningitis). The footnote gave the context*1. David Hume defined the concept of Causation as an inferred mental relationship, not a physical thing*2. Correlation does not prove Causation. If you don't agree, you can argue with Hume. From that philosophical perspective, the First Cause is an abstract concept, not a white-haired old Deity with a magic touch. :smile:


    *1. What does Hume say about causality? :
    Hume argues that we cannot conceive of any other connection between cause and effect, because there simply is no other impression to which our idea may be traced. This certitude is all that remains. For Hume, the necessary connection invoked by causation is nothing more than this certainty.
    https://iep.utm.edu/hume-causation/

    *2. Causation is a relation between objects that we employ in our reasoning in order to yield less than demonstrative knowledge of the world beyond our immediate impressions.
    https://iep.utm.edu/hume-causation/

    hqdefault.jpg
  • A first cause is logically necessary
    First, we should point out that, not only the first cause but any cause is supposed to be necessary.
    But this necessity kills causality itself: it's actually a problem in Spinoza's works that you probably already heard of. Since the cause cannot not produce the effect, it means the effect already lies in the cause somehow (and it means that time is a kind of illusion for Spinoza but that's another matter).
    But then: how can the cause produce an effect, since the effect already exists?
    Therefore, nothing can really be produced, and this kills causality. Or rather, it shows that causality is contradictory: causality can exist thanks to the absence of causality, and vice versa. That, of course, is a very short presentation of this subject (source: Brief Solutions to Philosophical Problems Using a Hegelian Method, Solution 10)
    LFranc
    David Hume addressed the philosophical Causation Problem by noting that, in Physics there is no Causation, only Change*1. Yet, the human mind attributes the Power of Causation (potential) to some unseen force. By the same reasoning, there are no Laws or Logic in the physical world. But the human mind seems to inherently "conceive" of consecutive Change as the effect of some prior physical input of Energy. It's a Belief, not a Fact.

    Hence, for both Physicists and Philosophers, Causation is logically "supposed to be necessary" --- to explain an observed Difference --- even when the original impetus is not empirically observable : e.g. First Cause of all change in the world. Therefore, I suppose that "the Effect lies in the Cause" in the sense of Aristotle's Potential. Which again, is a metaphysical concept, not a physical force or object. Likewise, Einstein's "space warped by gravity" is a metaphysical concept, which some imagine to be an empirical observation. Consequently, philosophers need not expect to have their belief about Causation confirmed by empirical evidence. We can only argue about the plausibility of the logical inference from difference, n'est ce pas?

    Therefore, in this thread, we are ultimately arguing about a metaphysical principle*2 to explain all changes in the world. So, if we track all physical changes back to the Big Bang, and stop, we have merely defined the First Effect, not the First Cause. :smile:


    *1. What does Hume say about causality?
    Hume argues that we cannot conceive of any other connection between cause and effect, because there simply is no other impression to which our idea may be traced. This certitude is all that remains. For Hume, the necessary connection invoked by causation is nothing more than this certainty.
    https://iep.utm.edu/hume-causation/

    *2. Metaphysical Primacy :
    Metaphysics is the branch of philosophy that studies the fundamental nature of reality. This includes studies of the first principles of: being or existence, identity, change, consciousness, space and time, necessity, actuality, and possibility.
    https://en.wikipedia.org/wiki/Metaphysics


  • A first cause is logically necessary
    ↪180 Proof indicates his prejudicial opinion that there can be nothing outside of space-time. {how do he know?} — Gnomon

    I did not claim or imply this.

    As I've stated in several of our exchanges, Gnomon, my metaphysical position more or less agrees with Spinoza's: there is no "outside of space-time" (or "beyond" with "possibilities") insofar as nature is unbounded in all directions (i.e. natura naturans is eternal and infinite) ... just as there is no edge of the Earth off of which one can fall, no north of the North Pole, etc.

    Stop making up sh*t. :sweat:
    180 Proof

    OK. I'll let you speak for yourself. Contrary to my interpretation, you're saying that "there can be something outside of spacetime?" Yes or No? After you have corrected my understanding of your metaphysical position on space-time vs eternity question, please answer the questions below.

    If I mis-interpreted your Immanentism position on the all-inclusive, no exceptions, expanse of space-time, I will apologize in this thread. But you would have to either reject the Big Bang theory outright, or explain the obvious implication of a time before space-time --- i.e. beyond the scope of scientific evidence. Unfortunately, unless you have slam-dunk & drop the mike evidence or argument, that discussion might require a new thread of its own. In any case, your earthbound "north of north pole" argument*1 is irrelevant to modern philosophical & cosmological conceptions of Space-Time.

    Do you think Stephen Hawking was "making up sh*t" when he said that "Time . . . had a beginning"*2? Do you accept that a sequence with a beginning must have an end --- not to mention a First Cause? Do you agree that beginnings & endings are set boundaries, that logically imply possibilities beyond the boundaries*3? Was Hawking spreading "woo-woo"?

    My metaphysical position also generally agrees with Spinoza . . . but with one scientific objection : in the 17th century, he plausibly assumed that "deus sive natura" was eternal, hence unbounded. But 20th century science found evidence to contradict that presumption*4. Which concept of "space-time" is compatible with your "metaphysical position" : 17th or 20th century? Again, I ask {how do he know?} :smile:


    *1. How valid is "What is North of North Pole?" argument? :
    "It is not an argument. It is an (imperfect, as always) analogy." ___Victor Toth
    https://www.quora.com/How-valid-is-What-is-North-of-North-Pole-argument

    *2. The Beginning of Time :
    The conclusion of this lecture is that the universe has not existed forever. Rather, the universe, and time itself, had a beginning in the Big Bang, about 15 billion years ago. ___Stephen Hawking
    https://www.hawking.org.uk/in-words/lectures/the-beginning-of-time

    *3. Did spacetime start with the Big bang? :
    In particular Roger Penrose has developed a view that the period since the Big Bang should be called an aeon, and that there were earlier aeons each infinitely long. This makes the Big Bang a kind of transition period between two aeons.
    https://physics.stackexchange.com/questions/5150/did-spacetime-start-with-the-big-bang
    Note --- This is not my view. Simply an expert's opinion/conjecture that the BB had a precursor. He may be "making up sh*t", but he has pristine math to back it up.

    *4. Immanence in Space & Time :
    As a prime example of a transcendent conception of space in this paper, Isaac Newton’s theories of space will be discussed as well as the mathematical framework within which Newton developed his physics: Euclidean space. On the other hand, as a role model for an immanent conception of space, Einstein’s general relativity will be analyzed as well as the geometry that lies behind this theory: Riemann’s differential geometry. . . .
    Unlike transcendent conceptions of space, in which space provides a super-structure for the organization of bodies and events that change over time, immanent conceptions of space do not rely on the presumption that space exists prior to bodies.

    https://www.performancephilosophy.org/journal/article/view/146/262
  • A first cause is logically necessary
    This is what I think 180 Proof failed to understand. He's an intelligent person, but I believe was convinced the argument was trying to say something it wasn't. The major struggle I've had in this OP was getting people break free of the"first cause is a God" argument that has been locked in debate for decades. It can be hard to shake for some. My hopes were to get both atheists and theists to see that we're missing an incredible point in the midst of the overwhelming concern about proving/disproving deities.Philosophim
    In his smirking reply to my post above --- "possibilities that go beyond space-time" --- indicates his prejudicial opinion that there can be nothing outside of space-time. {how do he know?} That working hypothesis may be necessary for the purposes of Empirical science, but it is self-limiting for the explorations of theoretical Philosophy. That would be like Columbus assuming the conventional belief of the era, that there is nothing over the horizon to the west of Europe.

    That space-time-is-all presumption may be a convenient position for a confirmed Immanentist, but may also be a self-imposed blindfold for someone who is not so sure that what-you-see-is-all-there-is. Ironically, what 180 is missing, due to his no-god prejudice, is that the OP says nothing about going beyond the bounds of space-time to find a First Cause. It even specifically warns against "Infinite causality", which some might identify with a biblical god. So, I'd be interested in his astute argument for-or-against the notion of a logically necessary First Cause, for a chain-of-events that is integral-with and directly-connected to the physical sequences of space-time*1. In other words, a Self-Caused sequence, or Spontaneous Generation. Did our world have a First Cause or merely a First Step?*3

    Many cosmologists have concluded that our bubble of space-time is bounded by a mysterious energetic beginning and an evanescent entropic ending*2. Some have postulated that the Big Bang was not an explosion from nothing into something, but merely an expansion of some pre-existing matter squished into a dimensionless mathematical Singularity. So, the experts disagree on the necessity for a First Cause, versus a resurrection from a previous incarnation of an un-caused (self-existent) eternally-cycling Multiverse. The M-verse has some basic characteristics of a traditional god, but presumably it's merely a cosmic mechanism without Consciousness or Entention. Sort of like a driverless taxi.

    Is it possible to ontologically understand the Big Bang theory without considering a context of "possibilities that go beyond space-time"? :smile:



    *1. Big Bang Self-Caused?
    The Big Bang was the moment 13.8 billion years ago when the universe began as a tiny, dense, fireball that exploded. Most astronomers use the Big Bang theory to explain how the universe began. But what caused this explosion in the first place is still a mystery.
    https://www.amnh.org/explore/ology/astronomy/how-did-the-universe-begin

    *2. Beginning of Time :
    The conclusion of this lecture is that the universe has not existed forever. Rather, the universe, and time itself, had a beginning in the Big Bang, about 15 billion years ago. ___ Steven Hawking
    https://www.hawking.org.uk/in-words/lectures/the-beginning-of-time

    *3. Space & Time are Matter & Energy :
    That makes Matter = Energy; Energy = Space; Space = Time. Therefore matter, energy, space and time are all interchangeable characteristics, which implies strongly that they are all forms of one thing.
    https://alasdairf.medium.com/are-matter-energy-time-space-all-interchangeable-e2dbf7d411e5
    Note --- Space-time is not a real thing, but the concept entails both Matter and Energy. Without Matter, there is no Space. Without Change, there is no Time.
  • A first cause is logically necessary
    Handshakes across the aisle.ucarr
    Thanks. Speaking of philosophical aisles :


    It seems to me your argument misses a significant distinction: 'that there is first cause' & 'what the first cause is'; "there is no limitation on what the first cause is', not in reference to 'that there was a first cause'. — 180 Proof
    I'm now expressing big gratitude to 180 Proof. He's done a superb job fulfilling my request. I now believe his statement above detects a fatal flaw in my argument. Philosophim has claimed there is no limitation on what a first cause can be. At the opposite end of the spectrum, he has claimed there is a conclusive limitation on that a first cause can be: logical necessity.
    ucarr
    is much more knowledgeable of Philosophy than I am. But his worldview & belief system (Immanentism ; p-Naturalism) has an inherent limit that precludes consideration of some logical possibilities that go beyond space-time : his "conclusive limitation". I suspect that you might agree with that physical barrier, while disagreeing with the implied logical limitation : Abstract Reason can go (in imaginary scenarios) where no material body can go. The human Mind can project (fantasy or logic) into the Future and into the Past, in order to learn about otherwise unknowable possibilities : e.g. Arthur C. Clarke, 2001 A Space Odyssey*1.

    In this thread, I never got the impression that you were arguing for any specific kind of First Cause (What), but merely reasoning about the logical necessity for something to kick-start the chain of Causation (That). The OP poses the question in generic (X or Y) & abstract (infinite prior causality) terms. So, I don't think 180's "distinction" really applies to this thread. He may be filling-in the "X" with a god-model of his own imagination.

    When says that there is "no limit" on what the Cause of Being might be {see PS below}, he's merely admitting that we are speculating about a state & event that is empirically unverifiable (no known rules), but logically plausible (rules of reasoning) : for example -- the Multiverse conjecture tries to have it both ways : Eternal Laws of Nature, and Unlimited Causal Energy (i.e. no Entropy).

    Nevertheless, for the purposes of an amateur forum, we can reasonably conclude that a contingent world (big bang beginning) requires a prior Cause of some kind (infinite ; recursive?), without taking the next step of identifying specific characteristics (loving, merciful?) of that Cosmic Causal Potential*2. However, space-time does place physical limits on "how causality functions". So, most causal conjectures are eventually forced to go beyond the physical barrier into the realm of Pure Reason .

    Some anti-first-cause arguments attempt to refute "Logical Necessity" and Metaphysical Necessity with alternative definitions and modalities. But Philosophim also offers Mathematical Necessity. So, take your choice : Physical Limits, Logical Limits, Mathematical Limits, or anything goes. All human reasoning has inherent limits, beginning with our physical senses and motives. :smile:


    *1. 8 predictions Arthur C. Clarke got right decades ago
    https://www.cnet.com/pictures/8-predictions-arthur-c-clarke-got-right-decades-ago-pictures/3/

    *2. "Because there are no other plausibilties to how causality functions, the only {logical} conclusion is that a causal chain will always lead to an Alpha, or first cause". — Philosophim

    *3. "I'm a p-naturalist¹ and thereby speculatively assume that aspects of nature are only explained within – immanently to – nature itself by using other aspects of nature, which includes "consciousness" as an attribute of at least one natural species." ___180 Proof
    https://thephilosophyforum.com/discussion/comment/867837
    Note --- The First Cause speculation is not about any particular "aspect" of Nature, but about all aspects of Nature : the Cosmos as a whole living (dynamic, if you prefer) system that was born and is fated to die. Who or What caused this system of Causation?


    PS___ 2 years ago : If a first cause is necessary
    " Understood, but my argument counters that. If a first cause is logically necessary, it is not necessary that it be a God, because a first cause is not bound by any prior rules of causality for its existence." — Philosophim
    I agree. That's why I refer to the philosophical Principle of First Cause or Necessary Being by various alternative names, including "BEING". But most people would equate those names with their own notion of "God". Which is why, for a while I spelled it "G*D", in order to indicate that it's not your preacher's notion of deity. Instead, it's what Blaise Pascal dismissively called "the god of the philosophers". Others call it simply "the god of Reason". That's what's left when you strip Religion of its traditional mythology & social regulations & emotional commitments. The power-to-exist is essential to living beings & non-living things, and is fundamental to philosophical discourse. It's the unstated premise of every assertion about what-is. So, I try to deal with the elephant-in-the-room head-on, instead of pretending it doesn't "exist" in conventional reality. ___Gnomon
  • A first cause is logically necessary
    You have said: "... before first cause, nothing."
    How do your descriptions of the inception of first cause have anything to work with other than nothing?
    ucarr
    I think you are sincerely trying to grasp an Idealistic worldview*1 that is radically different from your own Materialistic worldview*2. {pardon the pigeon-holing} Both are Metaphysical concepts created in philosophical Minds. Each perspective has developed a peculiar vocabulary of its own. So, you may think that Gnomon's worldview is Idealistic (no thing), and in direct opposition to Materialism (no thought). But my Enformationism worldview is not so easy to pigeonhole, because it is moderated by the Holistic BothAnd approach to understanding the Things of Reality and the Non-Things of Ideality.

    Application of the BothAnd Principle*3 requires one to look at both sides of any Either/Or argument as-if they are merely qualitative aspects of an inclusive holistic comprehensive worldview. But it doesn't mean that you have to ultimately accept one side or another. Instead, its goal is 3D stereoscopic vision : attempting to approximate a god-like understanding of Everything Everywhere All-At-Once, yet without the power of omniscience. So, that ballpark conjecture may appear to straddle the conceptual gap between the polar opposites, like the Colossus of Rhodes.

    For example : I just read this passage in Bernardo Kastrup's Science Ideated : "What seems to be beyond Coyne's ability to comprehend is that the dualism between mind and matter he implicitly relies on . . . . doesn't exist. To an idealist like me, there is no brain or matter outside or independent of mind. Instead, the 'material' brain is merely the extrinsic appearance, in some mind, of the inner mentation of (some other) mind." I understand the words, but I cannot imagine that bodies & brains are imaginary --- unless the image is in the Mind of God, which is itself a recursive idea in my mini-mind. So, it's also beyond my ability to imagine a world in which my own body is imaginary*4. My power of abstraction is not that omniscient.

    Therefore, I can't grok Exclusionary Materialism (no mind) or Absolute Idealism (no matter). My body & brain seem to automatically "see" the world in terms of material phenomena. Yet, my brain-functions are able to Abstract the matter away, and to treat its logical structure (noumena) as-if it is a real thing. So, I can only make sense of that Metaphysical Duality by reminding myself that the "Map is not the Terrain". From that perspective, I can enjoy the pristine idea of a Mind, and the messy reality of a Brain. And I can imagine an abstract-logic First Cause, without leaving my idea-causing Brain behind. :smile:

    Note --- The hypothetical "as-if" means an imaginary situation or a situation that may not be true but that is considered likely or possible.

    *1. Idealism :
    As an analytic idealist, Kastrup proposes that consciousness is the ontological primitive, the foundation of reality.
    https://danielpinchbeck.substack.com/p/analytic-idealism-a-revolutionary

    *2. Materialism :
    Materialism is a form of philosophical monism which holds that matter is the fundamental substance in nature, and that all things, including mental states and consciousness, are results of material interactions of material things.
    https://en.wikipedia.org/wiki/Materialism

    *3. Both/And Principle :
    # My coinage for the holistic principle of Complementarity, as illustrated in the Yin/Yang symbol. Opposing or contrasting concepts are always part of a greater whole. Conflicts between parts can be reconciled or harmonized by putting them into the context of a whole system.
    # The Enformationism worldview entails the principles of Complementarity, Reciprocity & Holism, which are necessary to offset the negative effects of Fragmentation, Isolation & Reductionism. Analysis into parts is necessary for knowledge of the mechanics of the world, but synthesis of those parts into a whole system is required for the wisdom to integrate the self into the larger system. In a philosophical sense, all opposites in this world (e.g. space/time, good/evil) are ultimately reconciled in Enfernity (eternity & infinity).
    # Conceptually, the BothAnd principle is similar to Einstein's theory of Relativity, in that what you see ─ what’s true for you ─ depends on your perspective, and your frame of reference; for example, subjective or objective, religious or scientific, reductive or holistic, pragmatic or romantic, conservative or liberal, earthbound or cosmic. Ultimate or absolute reality (ideality) doesn't change, but your conception of reality does. Opposing views are not right or wrong, but more or less accurate for a particular purpose.
    # This principle is also similar to the concept of Superposition in sub-atomic physics. In this ambiguous state a particle has no fixed identity until “observed” by an outside system. For example, in a Quantum Computer, a Qubit has a value of all possible fractions between 1 & 0. Therefore, you could say that it is both 1 and 0.

    https://blog-glossary.enformationism.info/page10.html

    *4. Cognitive dissonance occurs when a person believes in two contradictory things at the same time.

    IS IT POSSIBLE TO BELIEVE (dueling) IMPOSSIBLE (incompatible) CONCEPTS?
    eb4314a975a6699d973398d012ff71df.jpg

    Colossus-of-Rhodes.jpg
  • A first cause is logically necessary
    Have you not agreed with Gnomon (above) that immaterial yet real concepts -- as distinguished from matter -- are useful for correctly understanding your thesis, and therefore pertinent to it?ucarr
    Ha! Gnomon is not conspiring with to get our "story" straight. We just happen to view the First Cause postulate as a plausible philosophical explanation for the existence of a contingent & sequential Reality, in which a new thing or event necessarily follows from a previous event. The prior thing or event is what we call the "Cause"*1 of the subsequent thing or event. How we articulate that notion may vary. But in general we both seem to agree with the reasoning of Plato and Aristotle. If that sounds like Idealism to you, then so be it. :joke:

    *1. What is Hume's theory of cause and effect? :
    Hume saw causation as a relationship between two impressions or ideas in the mind. He argued that because causation is defined by experience, any cause-and-effect relationship could be incorrect because thoughts are subjective and therefore causality cannot be proven.
    https://study.com/academy/lesson/the-metaphysics-of-causation-humes-theory.html


    By immaterial existence I mean an abstract concept -- or some such entity -- that inhabits the mind apart from matter. Have you not agreed with Gnomon (below) that concepts are immaterial and real?ucarr
    I suspect that the term "immaterial"*2 may mean something different to you than to Gnomon & Philosophim. For example : concepts & ideas are not "real" but ideal. We are not trying to say what an abstraction is "made of", because it's not a material object, and is not "made of" any physical substance.

    I know that conceptual abstractions, such as Souls or Selves*3, do not fit neatly into the worldview of Materialism. But, regardless of their "true nature", they are useful concepts for philosophical understanding. And abstractions are essential for material technology*4. For example, the imaginary (as-if) notion of an Electrical or Quantum Field has allowed engineers to build cellular communication systems that work well, even though we don't know the "true nature" of the invisible mathematical relationships that constitute the so-called "Field". :nerd:

    *2. On the Meaning of "Immaterial" :
    Things we think of as immaterial, such as consciousness and soul, are material phenomena that we think must be immaterial because we do not yet know their true nature. To claim that something is immaterial implies it does not exist. Consciousness surely exists, and there are many good reasons to think souls do too.
    https://www.psychologytoday.com/us/blog/memory-medic/202103/the-meaning-immaterial
    Note --- I don't agree with this materialistic concept of "existence". Objects and Concepts "exist" in different "senses" : one is objective (sensory) and the other is subjective (ideational).

    *3. The Soul is a Self-concept :
    Self-concept is an overarching idea we have about who we are—physically, emotionally, socially, spiritually, and in terms of any other aspects that make up who we are.
    https://positivepsychology.com/self-concept/
    Note : I tend to use the secular concept of a "Self" to replace the religious concept of a "Soul". Neither is a material object, but a summation of all properties & qualities of a "Person", which is another abstraction. Hence, one abstraction can be a "component" of another concept, but you can't make anything physical from a pile of abstractions.

    *4. Abstractions in Science & Technology :
    Abstraction is an integral part of computational thinking and problem solving. It is also one of the most difficult parts of computational thinking to conceptualize. Much of this difficulty has to do with the semantics of the word “abstraction,” which is often inferred to mean unclear or vague. However, the more relevant definition of abstraction as it pertains to computer science is “the summary of something” or “the extraction from something.” . . . .
    Abstraction, as used in computer science, is a simplified expression of a series of tasks or attributes that allow for a more defined, accessible representation of data or systems. In computer programming, abstraction is often considered a means of “hiding” additional details, external processes and internal technicalities to succinctly and efficiently define, replicate and execute a process.

    https://www.learning.com/blog/examples-of-abstraction-in-everyday-life/
  • A first cause is logically necessary
    So, our world is an eternal following-causal-chain in the sense that its origin, Prime Cause, is an eternal logical necessity.ucarr
    I could agree with that statement, except that the "eternal" adjectives could be mis-interpreted. AFAIK the "causal chain" is spatial & temporal, not eternal : AFAIK, space-time began with a bang. The "logical necessity" is a concept in my mind, to explain the existence of the space-time world. It may be "eternal", but all I'm saying is that it is necessarily pre-big-bang. :smile:
  • A first cause is logically necessary
    Are you saying unique First Cause is necessary to chain of causation it's outside of and affecting?
    Can you elaborate additional details about the unspecified whereness -- positionally speaking relative to the whole -- of abstract-but-necessary Principle?
    ucarr
    Are you saying: a) the logical first cause has no material physicality; b) the logical first cause that has no material physicality exists within our universe?ucarr
    Yes. My understanding of a logically necessary First Cause is a philosophical conjecture, not a scientific observation. So there is no "whereness" to specify. You can call it simply a Philosophical Principle, or a god, as you wish ; but like all fundamental Principles, the Prime Cause is a theoretical Concept, an Idea with "no material physicality". However, the referent is not an anthro-morphic deity located in space-time, but more like the Abstract Rational Principle of the US founding fathers, and the European philosophers such as Leibniz and Thomas Paine*1.

    You can find more "details" under the heading of The Cosmological Argument*2. Colloquially, Deism is known as the "God of the Philosophers". As I said in the previous post : "But one sticking point seems to be confusing a logical First Cause (of some resulting chain of events) with an objective Thing or God operating in space-time". Philosophically, that confusion could be called a conflation of concepts & objects, of Ideality and Reality.

    The scientific Big Bang theory understandably avoided the philosophical question of where the Energy & Laws of Nature came from. That's because those logical necessities for a Chain of Causation are presumably Eternal & Everywhere. Once you have Potential & Algorithms, the manifestation of Matter is a sequential physical Effect of a singular metaphysical Cause. So, it would be less confusing to call the prerequisites for a physical causal world a universal Philosophical Principle instead of a particular Scientific Fact. :smile:


    *1. What is the deism theory of God? :
    Rather, deism is the belief in a sole creator god who set the universe in motion according to nature's laws and then left it to run on its own. The evidence of a creator is discernible through human reason and logic and has nothing to do with any scriptural authority, revelation, or miraculous events.
    https://study.com/academy/lesson/deism-the-founding-fathers-definition-beliefs-quiz.html

    *2. Cosmological Argument :
    Though they often disagreed, one principle of philosophy on which Plato and Aristotle agreed was that existence and the universe required a First Cause or Prime Mover - a god of some kind. Their argument was basically as follows. Every finite and dependent being has a cause. . . . .
    https://lah.elearningontario.ca/CMS/public/exported_courses/HZT4U/exported/HZT4UU02/HZT4UU02/HZT4UU02A04/_ld1.html
  • Is our civilization critically imbalanced? Could Yin-Yang help? (poll)
    Is our civilization critically imbalanced? How could applying Yin-Yang concepts help?
    (or… ancient philosophy to the rescue?!?)
    0 thru 9
    I must answer yes, but . . . . the imbalance seems to be mostly due to top-heavy technological power of humans over the rest of the world. When humans were upright apes, they had little advantage over plants & animals for making a living in the world. But, as their big brains began to solve fitness problems with artificial products instead of innate tools, they applied that leverage to out-compete most other animals. For much of history, that advantage was considered a good thing for humanity.

    Today though, the technology vs nature relationship is trending toward "critical imbalance" : what Malcom Gladwell called The Tipping Point*1. He wasn't talking about an the immanent end of the world, but about how complex & chaotic systems of all kinds can suddenly go off the rails : otherwise known as the Butterfly Effect. So, the term may also apply to the rapid changes in the physical and socio-cultural environment. The positive takeaway from his analysis is that understanding how things go awry is the first step toward a social or technical solution.

    On a more personal level, your suggestion of a complementary Yin Yang approach to restoring the balance of Nature and Culture may be the best that Philosophy can do to "rescue" the world from descending into permanent crisis mode --- as portrayed in the stream of Apocalyptic movies coming out of Hollywood --- where humans are faced with an existential struggle-for-survival. For Science & Technology, the related notion of Holism*2 is now being applied to complex systems that are hard to understand and control.

    Again, understanding how things go wrong, may help us to set them back on the right track. Without abandoning our technological advantage over a universe that is not designed for human thriving : is thrival better than mere survival? :smile:


    *1. The Tipping Point :
    Gladwell defines a tipping point as "the moment of critical mass, the threshold, the boiling point."
    https://en.wikipedia.org/wiki/The_Tipping_Point

    *2. Holism in science :
    Holism in science, holistic science, or methodological holism is an approach to research that emphasizes the study of complex systems.
    https://en.wikipedia.org/wiki/Holism_in_science
  • Types of faith. What variations are there?
    How many types of faith are there? Is religious faith special in that what is believed is taught systematically to many and reinforced versus faith an individual can have based on their own observations of how things tend to play out?TiredThinker
    Basically, there are only two types of Faith : Familial Trust or Rational Belief. All of us take certain things for granted, based on either a> repeated personal experience or b> other's experience via hearsay. Of course, type a> is specific and limited to the senses of a single person, and is implicitly accepted as true*1. But type b> is more general and combines the broader more-inclusive knowledge of many people, who may range from reasonable to irrational. However, Type a> may be expanded to include b> when defined as those of "like precious faith"*2. Since we have no way to verify those varied hearsay experiences, our acceptance of Reported Facts requires Trust in the Veracity & Authority of the source of information. Hence, the development of Scriptural Authority and Empirical Science.

    The inherent uncertainty of limited personal experience, may be why the gossip-grape-vine (including modern Mass Media) is so important to most of us. But also why Blind Trust can leave us mis-informed. Apart from sensory illusions, we have little reason to doubt our own direct impressions of what is real & true. But, gossip combines the feelings & opinions of several observers, resulting in diluted quality of scrutiny. So, any form of mass observation needs to be filtered through a rational screen to weed-out extreme subjective views in favor of the moderate average, which, for unverifiable philosophical questions*3, may be closer to Objective Truth.

    For those who are uncertain of their own rational powers, Familial Trust may be projected onto a Virtuous Person (Reverend), or Technically-trained Person (Scientist), or Authorized Scripture (Bible). But for Philosophers, trust in one's own personal reasoning abilities is essential to informed & vetted beliefs. :smile:

    Familial Trust : unquestioned Self-confidence or Social Trust (extended circle of familiarity)
    Rational Belief : tested & verified facts that are flexible, non-extreme, and logical.
    Hearsay : information received from other people that one cannot adequately substantiate; rumor. Social media.
    Grapevine : informal communication spreads throughout the organisation in all directions without following the formal path of communication. Social media.

    *1. Chico Marx in Duck Soup :
    "Well, who ya gonna believe me or your own eyes?"
    https://quoteinvestigator.com/2018/07/31/believe-eyes/

    *2. 2 Peter 1:1-8 King James Version : "Simon Peter, a servant and an apostle of Jesus Christ, to them that have obtained like precious faith with us"

    *3. Aristotle : "The virtue of justice consists in moderation, as regulated by wisdom."
    https://www.brainyquote.com/quotes/aristotle_148491
  • Are all living things conscious?
    I'm not sure I agree. But want to extend the discussion to you. If you think living things are "conscious" or aware or have a "me" from which they reference the world, does this apply to all living things? Or where is the cutoff point? And why?Benj96
    As others have noted, a philosophical discussion of Consciousness needs to be more narrowly defined than just basic chemical or neural Sentience. For example, the sensory ability to distinguish light from dark is an evolutionary advantage for many sub-conscious organisms. Hence, the emergence of light-sensing organs, mostly based on light activated chemicals such as Rhodopsin and Chlorophyll. Those sensations are the foundation of Feelings, but don't amount to Awareness-of-feelings until centralized by a brain. In that case, electrical neurons are necessary to channel sensations to the central processor, for sorting into Good or Bad For Me.

    Sapience is sentience developed into Intelligence, as in homo sapiens. But sapience requires some degree of Self-Awareness (a sense of Me) : a reference point upon which to base decisions that are in self-interest. For organisms with a physical Brain and metaphysical Self-Awareness, the next step is to develop a functional Mind. Mind is the basis of Intelligence, and according to Michael Levin : "all intelligence is really collective intelligence". That's because the brain merely coordinates the sentience of multiple sensory cells. And Evolution, since the emergence of single cell living organisms, seems to have been working for eons toward the sophisticated ability to know-what-you-know (Self-Consciousness).

    Scientific American magazine (Feb 2024) has an article entitled Minds Everywhere, which reports on recent research into cellular Cognition. The first example is a flatworm with no centralized brain, but with cells that can regenerate a head with eye-spots for light sensing. It can find food and avoid danger automatically, with no apparent sense-of-self. Although most plants have no eye-spots, they have leaves with chlorophyll-filled cells that perform a similar function --- in some cases to even move toward the light. A distributed-nucleus single-cell Slime Mold is not even as sophisticated as a plant, but without a brain or neurons, it can coordinate its oozing cells to move toward food, and to avoid danger. The article even reports an experiment in which a slime mold navigates a maze {image below}, requiring not just sensation, but learning. So, the emergence of Consciousness is a continuum, with no obvious cut-off point, such as homo sapiens.

    Based on such discoveries, Levin has concluded that "everything that's alive is doing this amazing thing" : Cognition. Which appears to answer the thread question in the affirmative. But it also indicates that just sensation & coordination & self-control does not amount to what we philosophers experience as Intelligent Consciousness. Which is a recent innovation of evolution, after billions of sol-years of groping in the dark, for slight advantages of fitness --- toward some future state that seems to require coordinated Complexity & Self-Awareness & Intelligence : a Person, not a Thing. :smile:


    SLIME MOLD NAVIGATING A MAZE and avoiding a hazardous obstacle
    2mJr6ZQ-asset-mezzanine-16x9-wRsya0F.jpg
  • Thomas Ligotti's Poetic Review of Human Consciousness
    ENOAH was pretty right on with his summary here: https://thephilosophyforum.com/discussion/comment/882856
    . Do you have a response ENOAH?
    schopenhauer1
    I'm guessing that 's response to Hannah Arendt's quote is just prior to my post above : "The way to transcend our mundane and fictional selves, the troubled self, the self of Consciousness, is to be the Real and organic self, the aware-ing Body simply in its Organic Living".

    Sounds like Sartre's concept of Authenticity : "To be authentic is to be clear about one's own most basic feelings, desires and convictions, and to openly express one's stance in the public .." Which is similar to my own notion of reasonable Self-awareness in a social context, in which case an "introspective person can rationally balance selfishness with selflessness". Of course, we could argue endlessly about the meaning of an "organic self". :smile:
  • A first cause is logically necessary
    I'm only going to tweak this a bit for clarification. You may not be implying this, I just want to be clear that a first cause as proven here is not outside of our universe, but a necessary existent within our universe. The balls on the pool table are not separate from the pool shooter. The entirety of the interaction is part of the universe.Philosophim
    My Poolshooter analogy was intended to illustrate that the Initial Cause was a separate sub-System outside the sub-system affected. Not necessarily outside of the known universe. Unless, there are no other (isolated) physical sub-systems, in which case the causal effects would apply to the whole universe, without exception. And the First Cause would have to be Meta-Physical (i.e. not subject to physical laws).

    So, if we are assuming that the chain of causation applies everywhere in the interconnected universe, then your immanent Cause could be its own Effect. For example the Cue ball is on the table, and can be impacted by the 8 ball. That's why my unique First Cause, or Causal Principle, is assumed to be off the table, outside the system affected.

    However, some have postulated that, in a Multiverse of multiple self-contained cause & effect systems, our local 'verse was impacted by another verse, causing the Effect we call the Big Bang. But of course, evidence for an eternal chain of 'verses is unavailable from inside our own system. So, I prefer not to specify where the imaginary Poolshooter is standing, and just call him an abstract-but-necessary Principle. :smile:
  • Thomas Ligotti's Poetic Review of Human Consciousness
    So I guess a question I can pose here, with all this in mind, is can anyone else see the validity in this idea of"excess" in existence, especially for the human experience? There is something that we are deluding ourselves in, with our goals, narratives, and ignoring of the situation, so that we don't have to "feel" or "sense" the excess. The excess might be akin to a certain sense of angst, existential dread, isolation, loneliness, ennui, and meaninglessness. Ligotti, used a term which is quite "horror" sounding with all caps- MALIGNANTLY USELESS. That might get at the feeling better.schopenhauer1
    From your description, it sounds like the implicit "excess" for humans is Self-Consciousness --- over & above basic consciousness. Without that talent for self-knowing, humans would be mere furless & fangless & clawless carnivores. Our behavior would be mostly innate & automatic & reasonless. Self-awareness allows humans to intentionally modify their behavior (biological drives) to suit their self-interest (goals ; narratives : aspirations), which may or may-not be in the interest of the community --- leading to law-breaking & treason, or to new standards of excellence. When fully immersed in the tribe, we could never feel "loneliness, ennui, or meaninglessness". So, from a pessimistic perspective, I suppose you could say that Selfishness is bad for humanity, but good for a person. But, what if that introspective person can rationally balance selfishness with selflessness?

    Contrary to Ligotti's horrors, Hannah Arendt, in The Human Condition, finds a more positive interpretation of "the essence of what it means to be human in a modern, increasingly alienated world". She seems to think that the trade-off of immersion in a community for isolated self-awareness can be worthwhile for those of a philosophical disposition. Only individuals can set their own goals, and construct a customized narrative or worldview. For example, independent-minded humans could ignore the tribal myths of Hitler, and see the "horror" in his insane "sanitary" purges. She's neither a romantic poet, nor a purveyor of horror stories, but a pragmatic philosopher. :smile:


    The Human Condition.
    "Human beings are unique in their ability to engage in thinking and reflection, which allows them to shape their own identities and find meaning in their lives."
    https://www.bookey.app/book/the-human-condition
  • Is maths embedded in the universe ?
    I will add that the expression that mathematics is 'in the world' is meaningless, just as it would be to say that a carton of eggs contains the number 12. Mathematics gives us a common symbolic means to describe, quantify and understand the world in a way that is not just based on individual perception but is grounded in a shared understanding and inherited knowledge.Wayfarer
    That summation should put an end to this thread. But of course, we can argue about the pertinent meaning of each word in the last sentence. The short answer is "Yes". But what do you mean by "in", or "embedded", or "grounded"? :wink:
  • Quantum Physics and Classical Physics — A Short Note
    In sum, quantum mechanics is a math based on limitations in measurements and probability. As you noted a "field" or "wave" is a mathematical entity that is often confused with a physical reality. Its a metaphor in English. When examining the ocean, we don't calculate every single molecule of water. Its unnecessary. Does that mean that ocean waves are not made up of molecules? No. But for what we're calculating, its easier. This is the same thing as measuring light as a wave vs as a particle. For some experiments and circumstances, its better to calculate light as particles than waves. Are waves of life comprised of particles? Of course. But in those circumstances in math, its just better to calculate it as a wave.Philosophim
    That's a good summary of the quantum quandary. The arcane math accurately predicts the results of chemical processes, but the "reality", of both the invisible particles and the intangible waves, is hard to imagine. As you noted, both are analogies to common sense experience on the macro level of reality. And even the notion of Entanglement may be simply an analogy to the well known Holistic functions of complex systems*1. Metaphorical analogies are too often "confused" with the Material objects they refer back to.

    The Santa Fe Institute*2 for the study of Complexity was established by quantum scientists (e.g. Murray Gell-Mann) among others, specifically to research the mysteries of complex physical systems, in which the contributions of parts may be subordinated to those of the system as whole. As you noted above, it's sometimes easier to calculate the behavior of a whole system than to track the zillions of dissolved parts (e.g. sea water & salt). The system label (e.g. cellular automata*3) may be an as-if metaphor to cover the interactive functions of uncountable physical elements.

    I mention the use of holistic methods by scientific experts, because my philosophical use of the term "Holism" on this forum is often confused with, and condemned as, New Age religious beliefs about unseen spiritual realities. Despite that Reductive defense against Holistic models, we need to emulate the quantum pioneers --- who at first were baffled by the counter-intuitive, and contra-classical, evidence of quantized systems and entangled behaviors --- but eventually got on with their job of revealing the underlying roots of Reality. :smile:


    *1. Holistic view in Complex Systems :
    By adopting a holistic perspective of complex systems, the system rule enables us to navigate the intricate interdependencies and dynamics within them.
    https://www.linkedin.com/pulse/system-thinking-rules-3-rule-holistic-view-complex-m/

    *2. Santa Fe Institute :
    . . . . the scientific knowledge that is associated with dynamic processes contains an amazingly rich variety of interconnections, involving distinct forms and levels of understanding. This variety of forms of dynamic knowledge, which is presently largely unrecognized, will be demonstrated via recent specific technical examples. It will be seen in these examples that the understandings (relationships) that have been discovered all have holistic characteristics. Moreover, their holistic qualities are unique in each case---in this sense, they are “emergent discoveries.” This suggests the future understanding of complex systems will involve the common activity of discovering new holistic forms of relationships.
    https://www.santafe.edu/research/results/working-papers/scientific-understanding-of-dynamic-phenomena-anal
    https://en.wikipedia.org/wiki/Santa_Fe_Institute

    *3. Automata :
    a moving mechanical device made in imitation of a human being.
    ___Oxford
  • Lucid Dreaming
    This is very pragmatic for me, it is just how things are, but I still get very excited with lucid dreams. Nowadays I never try to actively engage with the dreams, I just let them happen and observe.Olento
    The lucid dreams recounted in my previous post were pragmatic and intentional, because I had been consciously trying to solve the obstacles to flying silently like a bird. Human flight, since the Wright brothers, had always relied on noisy machines instead of innate muscle power. Today, electric flying machines with closed-loop propellers are getting close to the ancient dream of flying like a bird.

    In my not-so-ancient waking dream, while noiselessly flying above the tree-tops, I could hear the wind whispering, and sounds on the ground, not to mention seeing peripherally, without window-framing the view. These dreams had been inspired in part by the novel sensations of flying in my brother's single engine airplane, where the noise was intrusive, and made communication difficult, and the view was restricted.

    Some time later, and no drugs involved, I began to repeatedly dream that I could fly, more like superman, just by pure willpower. And it felt great --- no rackety engines or even flapping wings. In that case, I was no longer trying to solve pragmatic problems, just enjoying the feeling. Now, years later, I still have brief lucid dreams, mostly during a gradual wake-up. But I don't focus on details, just go with the flow. :smile:

    PS__ Years ago, I read an article about the use of Ayahuasca in South America, where indians imagined they could fly like a condor and prowl like a jaguar. Apparently, the drug stimulated something like a lucid dream, which was viewed as a spiritual reality by the drug user. Today, it's a recreational drug with spiritual implications, for those so inclined.

    Soul Quest Ayahuasca Church
    Orlando Florida
    https://www.ayahuascachurches.org/
  • What makes nature comply to laws?
    Perhaps it would be helpful to turn things around for a moment and ask, 'what would have to occur for nature to disobey laws?'unenlightened
    That's a good question. From our perspective as subjects to the Law, the physical regularities of Nature are Necessities*1 --- "gravity always wins". Also, since Nature has physical Forces to enforce those laws, the consequence is what we call Causation. Which raises the contentious question : is the lawful order & predictability of Nature due to top-down Causality (Lawmaker), or to fortuitous Accident (Chance)?

    A slight alteration of the OP might ask : why are these particular Necessities needed for the workings of Nature? If the mechanics of the universe was completely random, no physical path would be favored, and Evolution would not need to be Selective, and Statistics would never vary from a central norm. Obviously, the world we live in is mostly non-random, except perhaps on the quantum level, where spontaneity happens just enough to call it "Uncertainty". On the macro scale though, most processes are directional and predictable --- hence the "effectiveness" of Science. So, it seems that a bit of fundamental randomness is Necessary, only to allow degrees of freedom (flexibility) in the otherwise deterministic path of Evolution*2. The general direction, at least on Earth, is toward more complexity & organization, with just enough plasticity to allow for novelty along the way*3.

    Therefore, the universe --- or agents within --- could "disobey" natural Laws only if they were Un-necessary, or optional. But, as far as empirical Science can tell, the law-like limits on Physics are universal*4. And the only exceptions are found on the Quantum scale*5, which seem to serve only to dilute the mechanical rigidity of absolute Cause & Effect. So, only more Randomness, and less Lawfulness (i.e. Magic), would allow Nature to vary from it's legitimate path of orderly Causation & Evolution. :smile:


    *1. What does necessary mean in philosophy?
    In philosophy, necessity and sufficiency are two attributes that together constitute causality. A cause is necessary and sufficient to generate the effect. It being necessary is the negation or falsification: the effect cannot occur without the cause, so the cause is necessary for the effect to occur.
    https://www.quora.com/How-can-you-explain-necessity-in-philosophy

    *2. When being flexible matters :
    In this debate, it has been argued that our view of evolutionary causation should be rethought by including more seriously developmental causes and causes of the individual acting organism. . . . to reflect on the causal role of agency, individuality, and the environment in evolution.
    https://pubmed.ncbi.nlm.nih.gov/32285230/

    *3. Evolutionary Causation :
    Most scientific explanations are causal. This is certainly the case in evolutionary biology, which seeks to explain the diversity of life and the adaptive fit between organisms and their surroundings. The nature of causation in evolutionary biology, however, is contentious.
    https://philpapers.org/rec/ULLECB

    *4. Is natural law a law in the true sense? :
    Laws of nature are the only real laws based on principle and truth. Natural laws are universal, eternal, and immutable,
    Nature doesn’t “have” laws, since natural laws aren’t like man-made laws that tell people how they should behave. Instead, natural laws are merely our best descriptions of how we have observed that things behave within nature and how we think, by extension, things behave elsewhere within nature.
    https://www.quora.com/Is-natural-law-a-law-in-the-true-sense

    *5. Quantum Magic :
    Some quantum scale behaviors (e.g. tunneling) might seem magical, but they are never found on the macro level of reality.
  • What makes nature comply to laws?
    Quotes from this thread above :
    Would you agree that the average cosmic-trend-to-date has always been toward more local complexity (dust >> stars >> galaxies >> Earth), despite increasing general entropy {see image below}.Gnomon
    But I would say, in disagreement with the above
    "the average cosmic-trend-to-date has always been toward more local complexity (dust >> stars >> galaxies >> Earth), despite because increasing general entropy.
    unenlightened


    No. "because" not "by cause". An explanation is not a cause of anything except, occasionally, understanding.unenlightened
    OK. But, if your reply above is not a "causal" explanation, how does it explain --- increase understanding of --- how local complexity could increase, in apparent violation of the Second Law of Thermodynamics? My footnote *2, describes a possible explanation --- given certain conjectures --- of how high-density stars could form even-though (despite) the uphill pull against the inexorable cosmic expansion trend toward lower overall density of matter*3. Ironically, it uses the counter-intuitive statistical notion of "Entropy Density"*4. Perhaps, instead of striking out "despite" in favor of "because", your explanation should insert "probably" or "possibly".

    The Second Law is usually taken to be inviolable, with the possible exception of a highly unprovable & improbable First Cause scenario, as postulated in Cosmic Inflation theory, when presumably lax pre-bang physics also allowed a violation of the speed limit of light*5. Technically, that mathematical creation story took place before our Universe existed ; so it's not about Physics, but Meta-Physics : Voila! instant universe from nothing ; indistinguishable from magic. It's not a physical causal explanation, because it assumes a mysterious Cause that no longer exists in the real world. :smile:

    PS___ Again, I apologize for pushing this esoteric Causation enigma, but it's a hobby-horse of mine.


    *2. Did the Universe have zero entropy when it first began? :
    The answer, perhaps surprisingly, is no. The Universe not only wasn’t maximally organized at the start of the Big Bang, but had quite a large entropy even at the earliest stages we can describe
    https://bigthink.com/starts-with-a-bang/universe-zero-entropy/

    *3. Entropy vs Density :
    When we think about the Universe in the earliest stages of the hot Big Bang, we’re imagining all the matter and radiation that we have today — currently spread out across a sphere some ~92 billion light-years in diameter — packed into a volume about the size of the world’s largest pumpkin. The Universe back at that stage was incredibly hot and dense, . . .
    https://bigthink.com/starts-with-a-bang/universe-zero-entropy/

    *4. What is the relationship between entropy and density?
    Density measures how closely the atoms are packed, whereas entropy measures the disorder or randomness. . . .
    The law of entropy ( the law which says, entropy always increases) is better read as “there is a high probability that entropy always increases”. It’s not physics, but probability that governs this.

    https://www.quora.com/What-is-the-diference-between-density-and-entropy-basic

    *5. Is cosmic inflation faster than light?
    Around 13.8 billion years ago, the universe expanded faster than the speed of light for a fraction of a second, a period called cosmic inflation. Scientists aren't sure what came before inflation or what powered it. It's possible that energy during this period was just part of the fabric of space-time.
    https://science.nasa.gov/universe/overview/
  • What makes nature comply to laws?
    I think the scientific presumption is that demons do not exist. If they did exist, they would be just the entities to impose laws on particles like political economists such that wealth/energy would accumulate rather than dissipate.unenlightened
    Obviously, the "demon" was a metaphor that Maxwell used to illustrate a physical phenomenon --- work without a worker --- that had no better explanation. It remains a puzzle for both scientists and philosophers*1. But the metaphor is still used, not to explain but to illustrate, various anomalies in science. For example, physicist Paul Davies' The Demon in the Machine, in which he identifies the "demon" with Causal Information. Could that be the mysterious "entity to impose laws"? :smile:


    *1. Maxwell's Demon is a way of demonstrating that the laws of mechanics are compatible with microstates and Hamiltonians that lead to an evolution which violates the Second Law of thermodynamics by transferring heat from a cold gas to a hot one without investing work. . . .
    Maxwell’s Demon is a thought experiment devised by J. C. Maxwell in 1867 in order to show that the Second Law of thermodynamics is not universal, since it has a counter-example. Since the Second Law is taken by many to provide an arrow of time, the threat to its universality threatens the account of temporal directionality as well. Various attempts to “exorcise” the Demon, by proving that it is impossible for one reason or another, have been made throughout the years, but none of them were successful.

    https://www.ncbi.nlm.nih.gov/pmc/articles/PMC7516722/#:~:text=Maxwell%27s%20Demon%20is%20a%20way,hot%20one%20without%20investing%20work.

    But I would say, in disagreement with the above
    "the average cosmic-trend-to-date has always been toward more local complexity (dust >> stars >> galaxies >> Earth), despite because increasing general entropy.
    unenlightened
    So, you think Entropy is a causal force, instead of merely a degree of disorder or uncertainty in a system, as defined by physicist Rudolph Clausius?*2 In a similar metaphorical sense, I called my own coinage of "Enformy" a counter-force to Entropy. That's not yet a scientific fact, but it's a useful way for philosophers to think about the "general trend" of the universe to go downhill, while in local pockets of organization, like planet Earth, the thermodynamic trend has been "violated" ; reversed toward Life and Order. :cool:

    *2. Entropy is the general trend of the universe toward death and disorder. . . .
    the degree of disorder or uncertainty in a system

    https://www.merriam-webster.com/dictionary/entropy

    This is Hegel's "geist", disguised in pseudoscientific language.unenlightened
    That's a good analogy. But I object to the "pseudoscientific" characterization. "Holism" was originally a scientific term to describe how Evolution works its natural "magic". But the term was adopted by New Agers, and rendered contaminated by its association with supernatural beliefs. Similarly, the term "Metaphysics" was originally a useful philosophical term to describe topics, such as Mind, that are not understandable from a reductive physical perspective. Today, scientists use the term "Systems Theory" as a disguise for their holistic research*3. :nerd:

    Systems Theory & Holism :
    Systems have common defining properties, such as hierarchical ordering, coupling, permeability, holism, emergence, equifinality, and homeostasis. Representing the broader systems perspective are several specific theories and perspectives, such as Weick's theory of organizing, communication network perspectives, ecological and evolutionary perspectives, and self-organizing systems theory. Systems theory has been extensively applied in research areas ranging from communication design and adoption of technology use in organizational operations to professional communication, health campaigns, and public relations.
    https://www.researchgate.net/publication/316283969_Systems_theory
  • What makes nature comply to laws?
    If one begins with maximal simplicity, there is nowhere to go but towards complexity. However, once complexity has evolved, it can devolve into more simple forms, and there are many examples,unenlightened
    I'm not sure we are talking about the same thing. My post referred to "Russell's statistical argument to explain Nature's regularities". Then I asked a philosophical (non-scientific) question : not how, but "why would a random, non-designed, process (e.g. Evolution or coin flipping) have a tendency to average-out extreme states into a law-like & predictable moderate position?".

    Here's a physical example : The behavior of gas-in-a-box (Maxwell's Demon) illustrates --- without explaining --- that natural-but-inexplicable trait of averaging the pressure, by moving particles from a demon-caused condensed state toward a more natural diffuse state : order to disorder, or energy to entropy. In between those extreme states the gases were free to move forward and backward. Even biological evolution allows change to move back & forth*1. Would you agree that the average cosmic-trend-to-date has always been toward more local complexity (dust >> stars >> galaxies >> Earth), despite increasing general entropy {see image below}. If so, the topical question could be rephrased as : why do physical systems tend to follow a middle-of-the-road course, toward more & more order, as they evolve? Moreover, why is the cosmos now in a moderate state of Entropy, which allows Life & Mind to emerge?

    Scientists have not been able to empirically determine an “underlying reason” for that “law-like behavior”, or for the simple to complex direction of natural Evolution. But some philosophers have speculated beyond the physical boundaries of Science --- to postulate a First Cause or Logos --- hoping to explain the Impetus and Intention behind such regularities in a universe that could otherwise be totally random and directionless. Even some professional scientists, Terrence Deacon, Paul Davies, Max Planck, Norbert Wiener, etc, have used the term “teleology”, not to explain, but to describe the lawful & directional forms of natural processes. So, isn't it reasonable for even fun-loving amateur philosophers like us to push the boundaries toward a Theoretical and Metaphysical answer to those Why questions.

    Regarding "maximal simplicity", I must suppose that would equate to minimum organization and max Entropy, as in the heat death (big freeze) of the universe. Which is the opposite of the Big Bang"s demonic (hot & dense) low Entropy*2 beginning. The article below, by physicist Ethan Siegel*2, implies that Evolution began in an almost perfectly ordered (superdense) state*3 like a Black Hole, from which there was "nowhere to go", but toward more internal freedom to change, and to organize into more complex systems. But, why not take the easy path, directly to complete Entropy, without the eons-long detour of incremental steps toward more & more organization? Instead, the BB theory describes the original state as a hot-dense Plasma, which is like a gas-in-a-box situation. For some unknown reason, a metaphorical Demon (Inflation???) moved all the particles into one side of the box, then opened the door to allow it evolve eventually & naturally into stars & galaxies & us. Hence today, we find "particles" of matter organized into upright bipedal creatures with big brains, who ask dumb Why? questions.

    Here's one amateur philosophical (non-scientific) speculation of a possible answer to the OP question of "what makes nature comply with its own inherent laws of nature" {my added bold}. Entropy alone would never even get to the original plasma state. So, is it reasonable that some countervailing inherent "force" or "law"*4 is responsible for enforcing the "regulations" of evolutionary organization? :smile:

    PS___Sorry to unload on you. I had a lot of momentum. :joke:

    *1. Can Species Evolve Backwards? ;
    Thus, penguins didn't "devolve," they simply adapted to their new environment, and in that particular case, that meant losing a feature that had previously been beneficial.
    https://www.sciencealert.com/what-happens-when-species-evolve-backwards-the-strange-science-of-devolution

    *2. Did the Universe have zero entropy when it first began? :
    The answer, perhaps surprisingly, is no. The Universe not only wasn’t maximally organized at the start of the Big Bang, but had quite a large entropy even at the earliest stages we can describe
    https://bigthink.com/starts-with-a-bang/universe-zero-entropy/

    *3. Zero Entropy :
    No entropy means no random motion in molecular level that means zero temperature that means zero heat energy that means zero possiblity for energy conversion that means "heat death of the universe" that means freeze of entire universe including all atoms and photons everything.
    https://www.quora.com/What-would-happen-if-there-is-no-entropy-in-the-universe

    *4. Enformy :
    In the Enformationism theory, Enformy is a hypothetical, holistic, metaphysical, natural trend or force, that counteracts Entropy & Randomness to produce complexity & progress. [ see post 63 for graph ]
    a. I'm not aware of any "supernatural force" in the world. But my Enformationism theory postulates that there is a meta-physical force behind Time's Arrow and the positive progress of evolution. Just as Entropy is sometimes referred to as a "force" causing energy to dissipate (negative effect), Enformy is the antithesis, which causes energy to agglomerate (additive effect).
    b. Of course, neither of those phenomena is a physical Force, or a direct Cause, in the usual sense. But the term "force" is applied to such holistic causes as a metaphor drawn from our experience with physics.

    https://blog-glossary.enformationism.info/page8.html

    MAXWELL'S DEMON COSMOLOGY : low entropy initial state ; high entropy final state
    0*W3B7yn50vDpdITq0.jpeg

    COSMOLOGY : What Demon placed the BB at the top of the energy/entropy curve?
    Natural evolution has a law-like gravity ride downhill from the demonic Normal position
    Big%20Bang%20Curve.jpg
  • What makes nature comply to laws?
    I think it's called "regression to the mean". If you toss a coin twice you might get heads twice, tails twice or one of each ht or one of each th. If you toss a coin a million times, you are almost certain to get within a hundred or so equal numbers of heads and tails, because 'chances are'.unenlightened
    I just Googled Bertrand Russell's statistical argument*1 to explain Nature's regularities, without recourse to a supernatural lawmaker. At first it seems to make empirical sense. But with afterthought, Nature still shows evidence of top-down statistical "laws"*2, begging the question of a Lawmaker or Regulator of Nature's "program", to direct its meandering median path, perhaps toward some future state.

    For example, why would a random, non-designed, process (e.g. Evolution or coin flipping) have a tendency to average out extreme states? Is there a mathematical "gravitational" force pulling events toward some middle course? To suggest that Nature tends toward moderation also raises Why questions. Physical Science has postulated dozens of hypothetical "Forces" to explain consistent physical behavior ; four of them deemed "fundamental" to physics. Even Aristotle described four Causes in nature.

    Yet again, why would such mysterious invisible causal pulls & pushes, with power over tangible matter, emerge within a non-directional randomized system?*3 Even "Chance" and "Chaos" are found to be lawful*4, and subject to arbitrary tugs toward the statistical median. So, Russell's argument merely redirects the question, pointing to the empirical predictable regularities of mathematics, instead of the hypothetical Great Mathematician*5, who defines what is Normal. :smile:


    *1. The Natural Law Argument :
    The laws of nature are of that sort as regards a great many of them. They are statistical averages such as would emerge from the laws of chance; and that makes this whole business of natural law much less impressive than it formerly was. . . .
    if there was a reason for the laws which God gave, then God Himself was subject to law, and therefore you do not get any advantage by introducing God as an intermediary. You have really a law outside and anterior to the divine edicts, and God does not serve your purpose, because He is not the ultimate law-giver.

    https://www.mit.edu/activities/mitmsa/NewSite/libstuff/russell/node3.html
    Note --- this just kicks the Lawmaker question farther back down the road toward an "ultimate" Reason-maker. Perhaps, Plato's LOGOS?

    *2. Empirical statistical laws :
    An empirical statistical law or (in popular terminology) a law of statistics represents a type of behaviour that has been found across a number of datasets and, indeed, across a range of types of data sets.
    https://en.wikipedia.org/wiki/Empirical_statistical_laws

    *3. Is God a Mathematician? :
    Nobel Laureate Eugene Wigner once wondered about “the unreasonable effectiveness of mathematics” in the formulation of the laws of nature. Is God a Mathematician? investigates why mathematics is as powerful as it is. ___Mario Livio, astrophysicist
    https://www.amazon.com/God-Mathematician-Mario-Livio/dp/0743294068

    *4. Laws of Chaos :
    Chaos theory has been developed from the recognition that apparently simple physical systems which obey deterministic laws may nevertheless behave unpredictably.
    https://www.sciencedirect.com/topics/earth-and-planetary-sciences/chaos-theory

    *5. Who says God is a mathematician? :
    Michio Kaku explains why he believes in an intelligent creator and describes God as a “mathematician” and his mind as “cosmic music.” “The final resolution could be that God is a mathematician,” says Kaku. ___ Michio Kaku, theoretical physicist
    https://bigthink.com/the-well/mathematics/
  • What makes nature comply to laws?
    Note that it is ""Transcendental Idealism" not "Transcendent Idealism". The CPR is not about "Transcendent Idealism", as this would lie beyond what the human can cognitively grasp and would move into the realm of the unknowable. Not only beyond human experience but also beyond human reason, because beyond the scope of empirical investigation. Included would be such concepts as God and the soul.RussellA
    That's what I suspected. But some critics seem to think Kant was talking about a supernatural Heavenly Realm, instead of a Hypothetical or Metaphorical state of perfection. Philosophical conjectures are often "beyond the scope of empirical investigation", but seldom beyond the range of rational analysis. Sadly, Philosophical Metaphors are all-too-often taken literally by those opposed to any preternatural implications.

    On this forum, "transcendence" seems to be a taboo trigger-word for fully-invested Immanentists --- one in particular --- to get on their high-horse. Ironically, in a practical sense, I could be pigeon-holed into an Immanentism (reality vs ideality) slot. But when theorizing, I feel free to go beyond the current state of empirical knowledge, and to speculate into the unknown. Yet some would dismiss that philosophical freedom as a religious commitment to a supernatural faith. :halo:

    Immanence and Transcendence :
    Both what we can know by reason (immanence) and what we can know only by revelation (transcendence) are reflections of the very being of God. By contrast, immanence would signify that human reason is the highest norm for our knowledge of ethical and religious practices.
    https://academic.oup.com/edited-volume/28079/chapter-abstract/212098546?redirectedFrom=fulltext

    The CPR is not about religion or the spiritual realism, but is about what we can practically know about the world using reason and observation.RussellA
    Thanks for the quote. As I indicated above, I assumed that Kant was writing as a reality-exploring philosopher --- searching for the boundaries of Epistemology --- not as a Christian apologist. However, some on TPF reject anything he says as-if it was religious propaganda. Yet he seems to rely on mundane reasoning, not on divine revelation, for his conclusion that there are some "things" (ding an sich) that are not accessible to "empirical investigation". And it's exactly those known-unknowns that intrigue me. :nerd:

    Kant uses such a Transcendental Argument in his Refutation of Idealism in B275 against the Idealism of Berkeley in order not to prove that things exist independently of the mind, but only the possibility that things exist independently of the mind.RussellA
    Obviously, it would be impossible to prove anything beyond empirical evidence or the reach of reason. But what difference does it make to assert the "possibility" of such ding an sich? I'm guessing that he was responding to some aspect of Berkeley's Idealism. Ironically, Kant's own philosophy has the label "Idealism" pinned on it. So, he's not rejecting the general concept of Meta-Physical Reality, but some particular detail of Berkeley's formulation. Yes? :cool:

    "Transcendental Idealism" uses the Transcendental Argument to make sense of the world given our sensory experiences.RussellA
    Thanks again. That makes sense to me. Although it obviously doesn't compute for some Kant bashers. Taken literally, the title "Transcendental Idealism" seems to be directly opposite to "Immanent Realism". Was that effrontery intentional? :smile:

    PS___ The OP seems to be questioning the possibility of a First Cause or Lawmaker to force Nature into compliance with somewhat arbitrary top-down "laws", as opposed to innate regularities emerging bottom-up, due to the constraints of random interactions. Top-down Laws would be Transcendent, while bottom-up Regularities would be Immanent. Hence, the thread's side-track into questioning Kant's notion of things & forces "beyond our sensory experience" or our "cognitive grasp".
  • What makes nature comply to laws?
    The term "transcendental idealism" should be thought of as a name rather than a description, as the Champs-Élysées is a name and not a description.
    In fact, he proposed renaming his transcendental idealism with the more informative name of "formal" or "critical idealism," (Introduction to Critique of Pure Reason)
    RussellA
    I'm not a Kant scholar, and have never read any of his works. But, "Transcendence" is inherently a debatable term, since it is based on subjective imagination instead of objective observation. Some critics seem to assume that Kant's "transcendental" referred to a religious heavenly realm of perfection isolated from the imperfect physical world. But others, such as the 19th century Transcendentalists, apparently believed in a parallel "spiritual" realm within this world, perceivable via intuition. For example, as depicted in movies : innocent children, guided by feelings instead of reason, can "see" dead people, or demons, or disguised alien monsters.

    Yet I'm getting the impression that Kant may have been merely making a pragmatic philosophical distinction between concrete physical Reality (actuality) and abstract mental Ideality (possibility), as a way to discuss metal phenomena (i.e. noumena), without the baggage of habitual physical preconceptions. But his choice of "Transcendence" as a label may sound absurd to those of a Materialist worldview, in which nothing could possibly transcend the apparent reality of the 5 senses --- as confirmed by empirical science. Unfortunately, his support for Christian doctrine would make his objectivity suspect.

    How do you interpret his usage of "transcendence"? Specifically, in his view, what limits are being surpassed? Was he denigrating mythical Pure Reason in favor of mundane non-magical Practical Intuition? :smile:


    Practical Intuition :
    Practical Intuition provides the tools you need to develop your intuitive potential to its fullest.
    https://www.goodreads.com/en/book/show/1242635

    Non-Magical Intuition :
    Intuition is a form of knowledge that appears in consciousness without obvious deliberation. It is not magical but rather a faculty in which hunches are generated by the unconscious mind rapidly sifting through past experience and cumulative knowledge.
    https://www.psychologytoday.com/us/basics/intuition

    PS___ In Philosophy Now magazine #158, editor Grant Bartley discusses the cover question : Freewill vs Determinism. In answer to "what freewill involves", he refers to Kant's notion of going beyond physical limits : "Kant calls the will 'transcendental' --- by which he means that for it to operate, will must not be part of the causal system of the physical world". In other words, "Transcendental" means Supernatural.

    Is that a valid interpretation of Kant? I'm sure even to mention such an "outrageous" possibility on TPF would cause to do his best ballistic Trump imitation : red face, sneering & blustering. But I'm seriously seeking to understand what Kant was talking about, because he's "one of the most influential philosophers in the history of Western philosophy". https://iep.utm.edu/kantmeta/
  • What makes nature comply to laws?
    Does this mean that transcendental idealism is in the end unavoidable and there is no realistic alternative to this world-view? And is the possibility and success of science proof, that Kant was rightfully claiming that we can never attain to a knowledge of things surrounding us per se i.e. independent of us?Pez
    I'm not a Kant scholar. But my understanding of his Transcendental Idealism*1 is that it's merely an admonition to idea-mongering philosophers, not to confuse our artificial worldviews with absolute Reality. This is not exactly claiming, like Berkeley, that our perception of the world sees only "appearances" that represent the Ideal world as-if objects (ding an sich) ultimately exist in the mind of God. But merely to note that humans see only superficial Properties, that are meaningful to our space-time physical needs, instead of essential eternal Qualities.

    Quantum Physics (QP) is another reminder that our 5 senses are attuned only to appearances, and our mental images represent only a fraction of reality. Heisenberg even echoed Kant to say that the "appearances" we call Reality, don't exist until observed. So, the quantum pioneers developed a new (non-mechanical) statistical worldview, to artificially sharpen the fuzziness of sub-atomic reality into a practical alternative to unrealistic Platonic Idealism.

    In other words, QP is an attempt to make such abstract digital mathematical information useful to us coarse concrete analog organisms. As you suggested, the "success" of Quantum Physics, so alien to Classical Physics, allows us to adapt our way of dealing with the "transcendental" aspects of the world to a novel counter-intuitive "knowledge of things" that are not really things, such as non-local waves that can also behave like particles .

    Whatever the Real World is, it is much more than our limited senses can cope with. So, we condense the incomprehensible behaviors of the Cosmos into mathematical symbols, and call them "natural laws". But Nature doesn't "comply" with our definitions ; instead, our formal laws are attempts to conform with Nature's regularities, symmetries, harmonies & proportions. :smile:


    Transcendental Idealism :
    In Kant’s view, human cognition is limited to objects that somehow depend on our minds (namely, appearances), whereas the mind-independent world (things in themselves) lies beyond the limits of our experience and cognition.
    https://iep.utm.edu/kant-transcendental-idealism/

    QUANTUM REALITY makes no sense to our senses
    79494-ggdmpugplg-1516284022.jpg